LSAT and Law School Admissions Forum

Get expert LSAT preparation and law school admissions advice from PowerScore Test Preparation.

User avatar
 Dave Killoran
PowerScore Staff
  • PowerScore Staff
  • Posts: 5852
  • Joined: Mar 25, 2011
|
#44166
Complete Question Explanation
(The complete setup for this game can be found here: lsat/viewtopic.php?t=9437)

The correct answer choice is (A)


If P is not selected, then O cannot be selected. Via the contrapositive, if O is not selected, then N and M cannot be selected. This information is sufficient to eliminate answer choices (B), (C), (D), and (E). Thus, by process of elimination, answer choice (A) is proven correct.

Get the most out of your LSAT Prep Plus subscription.

Analyze and track your performance with our Testing and Analytics Package.